LSAT and Law School Admissions Forum

Get expert LSAT preparation and law school admissions advice from PowerScore Test Preparation.

User avatar
 Dave Killoran
PowerScore Staff
  • PowerScore Staff
  • Posts: 5853
  • Joined: Mar 25, 2011
|
#47023
Complete Question Explanation
(The complete setup for this game can be found here: lsat/viewtopic.php?t=15947)

The correct answer choice is (D)

This is a List question, and in pattern games always be sure to do List questions first, in order to establish a workable hypothetical. The easiest rule to apply from a visual standpoint is the last rule, “No candidate can speak fourth at more than one of the meetings.” Answer choice (C) violates this rule and is thus incorrect. The next easiest rule to apply is probably the first rule. Answer choice (A) violates this rule (R does not appear) and is therefore incorrect. Finally, applying the second rule eliminates answer choices (B) and (E). Answer choice (D) is proven correct by process of elimination.

Get the most out of your LSAT Prep Plus subscription.

Analyze and track your performance with our Testing and Analytics Package.